AIR-02, Kanav Saluja Mock Test - 1

You might also like

Download as pdf or txt
Download as pdf or txt
You are on page 1of 53

MOCK COMMON LAW ADMISSION TEST, 2023-24

FREE CLAT MOCK

Duration : 120 minutes Candidate Name :_______________


Contact No. :
_______________
Max. Marks : 120 Date of Exam ________________
:

INSTRUCTIONS TO CANDIDATES

1. No clarification on the question paper can be sought. Answer the questions as they are.
2. There are 120 multiple choice objective type questions.
3. Each question carries ONE mark. Total marks are 120.
4. There is a negative marking of 0.25 marks for every incorrect answer.
5. Candidates have to indicate the correct answer by darkening one of the four responses
provided, with a BALL PEN (BLUE OR BLACK) in the OMR Answer Sheet.
Example: For the question, "Where is the Red Fort located?” the correct answer is
(b).
The candidate has to darken the corresponding circle as indicated below:
(a) Kolkata (b) Delhi (c) Jodhpur (d) Chennai
Right Method Wrong Methods

6. Answering the questions by any method other than the method indicated above shall be
considered incorrect and no marks will be awarded for the same.
7. More than one response to a question shall be counted as wrong.
8. The candidate shall not write anything on the OMR Answer Sheet other than the details
required and in the spaces provided for.
9. After the Test is over, the candidate has to return the OMR Answer Sheet to the
invigilator. The candidate should take the Test Paper along with them.
10. The use of any unfair means by any candidate shall result in the cancellation of his/her
candidature.
11. Impersonation is an offence and the candidate, apart from disqualification, may have to
face criminal prosecution.
12. Electronic gadgets like mobile phones, pagers or calculators are strictly not permitted
inside the Test Centre/Hall.
13. The candidates shall not leave the hall before the Test is over.

Page 1 of 53
SECTION 1 - ENGLISH

Directions (Q1-Q24): Read the passages carefully and answer the questions.

Passage 1(Q1-Q5):

Tiger conservation in India represents an excellent case study of the many challenges facing
conservation programs internationally. It is well understood that tigers are sensitive to human
disturbances and large areas of habitat need to be protected for their conservation. Such
protected areas in India are managed by the governments using an exclusionary approach.
However, this approach is known to create several issues with local communities, including
historical, legal, livelihood and management issues with a volume of literature suggesting the
inclusion of local communities in management. Yet, other evidence suggests that inclusion of
communities in tiger conservation may lead to anthropogenic disturbances that can jeopardize
tigers. The gravity of the situation is reflected in the recent disappearance of tigers from two
key protected areas in India, the Sariska and Panna Tiger Reserves. It is generally argued that
the exclusionary approach was brought into India by a group of “urban wildlife enthusiasts,
voluntary groups and preservation minded foresters” who won over “powerful, highly-
placed decision makers” and the most influential political leader, Prime Minister Indira
Gandhi in the 1960-70s and 1980s. The groups that pursued the establishment of PAs
included urban and foreign tourists, the elite section that equated charismatic species with
“national prestige”, international bodies working on conservation, certain sections of the
forest and wildlife departments, and biologists, who believed that species and pristine areas
ought to be protected for ‘science.’ These movements stressed the bio centric view (with
humans being merely one among the species) over the then predominant anthropocentric
view (where nature exists to be managed for human consumption). Since its implementation,
the critique of the exclusionary approach has burgeoned. In particular, ‘developing’ countries
have significantly different socio-economic and ecological contexts than the ‘developed’
countries, making the exclusionary approach unfit and undesirable in many cases. The
approach has been termed elitist and called an “ecologically updated version of the White
Man’s Burden” while being equated to ‘imperial domination’ when seen from the
perspectives of local communities. Further, the approach has been described as simplistic in
overlooking the intricate relationship between communities and their ecosystems.
Philosophically, the overarching criticism of the approach is that the real cause of general
environmental degradation is not the subsistence of the poor, but consumption by the rich,

Page 2 of 53
industrial and commercial pressures.

Q1) What is the main purpose of the passage?

a) Track the success of the tiger conservation programme in India


b) To examine the advantages of including local communities in tiger
conservation
c) To analyse the exclusionary approach of tiger conservation
d) Both (b) and (c)

Q2) Which of the following best describes the author’s opinion?

a) The author is in favour of the exclusionary approach


b) The author is opposed to the exclusionary approach
c) The author is neutral and does not have any inclination
d) None of the above

Q3) Which of the following statements about the exclusionary approach is least likely to be
inferred from the passage?

a) It is considered an elite phenomenon


b) It stresses the anthropocentric view over the biocentric view
c) It argues that wildlife areas should be protected for the purpose of
‘science’
d) The government of India follows the exclusionary approach

Q4) The word burgeoned is the antonym of:

a) Shrink
b) Frequent
c) Prosper
d) Rising

Q5) All these are criticisms for the exclusionary approach, except?

a) The exclusionary approach is unfit and undesirable for developing countries


b) It stresses, too much, on the importance of ‘science’ by advocating for protected

Page 3 of 53
areas.
c) It is seen as a form of imperial domination
d) It is an overly simplistic approach for viewing the relationship between the ecosystem
and local communities.

Passage 2: (Q6-Q10):

Every day, millions of shoppers hit the stores in full force both online and on foot searching
frantically for the perfect gift. Last year, Americans spent over $30 billion at retail stores in
the month of December alone. Aside from purchasing holiday gifts, most people regularly
buy presents for other occasions throughout the year, including weddings, birthdays,
anniversaries, graduations, and baby showers. This frequent experience of gift giving can
engender ambivalent feelings in gift-givers. Many relish the opportunity to buy presents
because gift-giving offers a powerful means to build stronger bonds with one’s closest peers.
At the same time, many dread the thought of buying gifts; they worry that their purchases
will disappoint rather than delight the intended recipients. Anthropologists describe gift
giving as a positive social process, serving various political, religious, and psychological
functions. Economists, however, offer a less favorable view. According to Waldfogel (1993),
gift-giving represents an objective waste of resources. People buy gifts that recipients would
not choose to buy on their own, or at least not spend as much money to purchase (a
phenomenon referred to as ‘‘the deadweight loss of Christmas”). To wit, givers are likely to
spend $100 to purchase a gift that receivers would spend only $80 to buy themselves. This
‘‘deadweight loss” suggests that gift-givers are not very good at predicting what gifts others
will appreciate. That in itself is not surprising to social psychologists. Research has found
that people often struggle to take account of others’ perspectives— their insights are subject
to egocentrism, social projection, and multiple attribution errors. What is surprising is that
gift-givers have considerable experience acting as both gift-givers and gift recipients, but
nevertheless tend to overspend each time they set out to purchase a meaningful gift. In the
present research, we propose a unique psychological explanation for this overspending
problem—i.e., that gift-givers equate how much they spend with how much recipients will
appreciate the gift (the more expensive the gift, the stronger a gift-recipient’s feelings of
appreciation). Although a link between gift price and feelings of appreciation might seem
intuitive to gift-givers, such an assumption may be unfounded. Indeed, we propose that gift-
recipients will be less inclined to base their feelings of appreciation on the magnitude of a gift

Page 4 of 53
than givers assume.

Q6) The process of gift giving has been described in the passage as one of the following
options, except?

a) A positive social process


b) A means to strengthen a relationship
c) An inexpensive way to show gratitude
d) A waste of resources

Q7) What is the tone of the passage?

a) Argumentative
b) Analytical
c) Narrative
d) Abstract

Q8) What best describes the meaning of the word ‘ambivalent’ as used in the passage?

a) Anxious
b) Unreasonable
c) Several
d) Conflicted

Q9) What statement is least likely to be inferred from the passage?

a) Gift-givers can hardly ever predict what the receiver wants


b) That there exists a link between gift price and feelings of
appreciation.
c) Gift serves many political and religious purposes
d) Some people have a fear of disappointing receivers of the gift

Q10) Social psychologists find the ‘deadweight loss’ a phenomenon that is:

a) Unrealistic
b) Impractical
c) Doubtful

Page 5 of 53
d) Predictable

Passage 3: (Q11-Q15):

Search engines have changed the way we use the Internet, putting vast sources of information
just a few clicks away. But Harvard professor of psychology Line Daniel Wegner’s recent
research proves that websites—and the Internet—are changing much more than the
technology itself. They are changing the way our memories function. Wegner’s latest study,
“Google Effects on Memory: Cognitive Consequences of Having Information at Our
Fingertips,” shows that when people have access to search engines, they remember fewer
facts and less information because they know they can rely on “search” as a readily available
shortcut. Wegner, the senior author of the study, believes the new findings show that the
Internet has become part of a transactive memory source, a method by which our brains
compartmentalize information. First hypothesized by Wegner in 1985, transaction memory
exists in many forms, as when a husband relies on his wife to remember a relative’s birthday.
“[It is] this whole network of memory where you don’t have to remember everything in the
world yourself,” he says. “You just have to remember who knows it.” Now computers and
technology as well are becoming virtual extensions of our memory. The idea validates habits
already forming in our daily lives. Cell phones have become the primary location for phone
numbers. GPS devices in cars remove the need to memorize directions. Wegner points out
that we never have to stretch our memories too far to remember the name of an obscure
movie actor or the capital of Kyrgyzstan—we just type our questions into Google. “We
become parts of the Internet in a way,” he says. “We become part of the system and we end
up trusting it.” Wegner concedes that questions remain about whether dependence on
computers will affect memories negatively: “Nobody knows now what the effects are of these
tools on logical thinking.” Students who have trouble remembering distinct facts, for
example, may struggle to employ those facts in critical thinking. But he believes that the
situation overall is beneficial, likening dependence on computers to dependence on a
mechanical hand or other prosthetic devices. And even though we may not be taxing our
memories to recall distinct facts, we are still using them to consider where the facts are
located and how to access them. “We still have to remember things,” Wegner explains.
“We’re just remembering a different range of things.” He believes his study will lead to
further research into understanding computer dependence and looks forward to tracing the
extent of human interdependence with the computer world—pinpointing the “movable

Page 6 of 53
dividing line between us and our computers in cyber networks.”

Q11) What is the main purpose of the passage?

a) To argue that the internet has made people dependent on it for basic
information
b) Show the findings of a study that examines the effect of technology on
memory
c) To discuss the effects on transaction memory in light of technology
d) To demonstrate how technology interferes with logical thinking

Q12) Technology has affected memory in these ways, except?

a) People remember fewer facts nowadays


b) Technology might interfere with critical thinking
c) People don’t have to remember a relative’s birthday on their own
d) It has made people a part of the internet and they trust it

Q13) What does “the extent of” in the last statement of the passage mean?

a) The scope of
b) The expanse of
c) The amount of
d) The stretch of

Q14) Which statement is most likely to be inferred by the passage?

a) Cell Phone and GPS have become transaction memory sources


b) Dependence on technology overall has a negative impact
c) We are not taxing our memory to memorize facts
d) We have started remembering a different range of things after the introduction of
technology

Q15) What according to the passage does Wegner want to prove?

a) That dependency on the internet has a positive effect


b) That reliance on the internet has a negative impact

Page 7 of 53
c) How far can the extent of human dependency on the internet reach
d) None of the above

Passage 4: (Q16-Q20):

As of May 1, 2020, COVID-19 had already infected over 3 million people and killed 250,000
people. Many political leaders and governments were criticized for their responses to the
pandemic. The US government was criticized for insufficiently testing its citizens during the
early stages of the crisis; the Japanese government was criticized for not declaring a national
emergency sooner than they did, in the hope of hosting the Olympic Games as planned;
medical doctors and nurses in Hong Kong went on strike due to government’s inefficiency in
handling the pandemic. All of these anecdotes suggest that world leaders should suffer in
their political ratings during this crisis, because many, if not all, countries have failed to
aggressively address this pandemic effectively. Indeed, even the best possible outcome from
COVID-19 would involve distressingly high numbers of infected and dead and a severe
impact on the economy, leaving political leaders vulnerable to bad publicity. Political science
literature, however, suggests another possibility. Specifically, the “rally ‘round the flag”
effect suggests that citizens tend to support their national leaders in times of international
crises. The rally effect can be explained by enhanced feelings of patriotism, under the
perception that one’s group is under attack and hence unity is required to defend the group
and support the incumbent. Identified in the 1970s, the rally effect has received empirical
support in many international crises. US presidents’ approval ratings increased during the
Cuban missile crisis, Iran hostage crisis, Gulf War, 9/11 terrorist attack, and the death of
Osama bin Laden. Multiple psychological theories converge to support this hypothesis.
System justification theory posits that people are motivated to justify and rationalize the way
things are, even when the social, political, or economic systems negatively affect their self-
interest. Perceived threat and powerlessness are key triggers of system justification beliefs,
leading people to support the status quo and display increased trust toward authorities such as
governments, because doing so reduces uncertainty and minimizes the perceived threat. Past
research finds that liberals and conservatives show the same system justification behaviors
and beliefs being exposed to threats. Research on cultural evolution provides an explanation
for this effect too. Cultural evolutionary models suggest that adherence to group norms and
support for group leaders can preserve group unity during times of threat and scarcity. In
particular, tightness−looseness theory suggests that group cohesion and coordination are

Page 8 of 53
necessary for human groups to overcome existential threats, and so it is functional for human
groups to rally around strong leaders in the face of these threats

Q16) What is the key theme of the passage?

a) To discuss the weak response of various governments to the Covid-19


crisis
b) How the rally effect has come up in face of the Covid-19 crisis
c) To highlight the impact of Covid on economic and political aspects around the
world
d) Both (a) and (b)

Q17) The following can be inferred about the rally effect, except?

a) One feels intense emotions of patriotism


b) Leaders usually benefit when the rally effect sets in
c) People tend to rationalize things that don’t even impact their self-interests in the face
of crisis
d) Threat, powerlessness and scarcity are the main reasons for the rally
effect

Q18) What does ‘empirical support’ as given in the passage mean?


a) Based on practical observations
b) Based on theory
c) Based on quantitative data
d) None of the above

Q19) What is the nature of the explanations presented by System Justification Theory and by
Cultural Evolutionary Models in relation to each other?

a) Contradictory
b) Identical
c) Complementary
d) Disjunctive

Q20) Which of these statements reflect the reasons for the rally effect?

Page 9 of 53
a) Reduces uncertainty and minimizes the perceived threat
b) Group cohesion and coordination are necessary for human groups to overcome
existential threats
c) Can preserve group unity and pro-sociality during times of threat and
scarcity
d) All of the above

Passage 5: (Q21-Q24):

Even then my only friends were made of paper and ink. At school I had learned to read and
write long before the other children. Where my school friends saw notches of ink on
incomprehensible pages, I saw light, streets, and people. Words and the mystery of their
hidden science fascinated me, and I saw in them a key with which I could unlock a boundless
world, a safe haven from that home, those streets, and those troubled days in which even I
could sense that only a limited fortune awaited me. My father didn’t like to see books in the
house. There was something about them— apart from the letters he could not decipher—that
offended him. He used to tell me that as soon as I was ten he would send me off to work and
that I’d better get rid of all my scatterbrained ideas if I didn’t want to end up a loser, a
nobody. I used to hide my books under the mattress and wait for him to go out or fall asleep
so that I could read. Once he caught me reading at night and flew into a rage. He tore the
book from my hands and flung it out of the window. “If I catch you wasting electricity again,
reading all this nonsense, you’ll be sorry.” My father was not a miser and, despite the
hardships we suffered, whenever he could he gave me a few coins so that I could buy myself
some treats like the other children. He was convinced that I spent them on licorice sticks,
sunflower seeds, or sweets, but I would keep them in a coffee tin under the bed, and when I’d
collected four or five reals, I’d secretly rush out to buy myself a book. My favorite place in
the whole city was the Sempere & Sons bookshop on Calle Santa Ana. It smelled of old
paper and dust and it was my sanctuary, my refuge. The bookseller would let me sit on a
chair in a corner and read any book I liked to my heart’s content. He hardly ever allowed me
to pay for the books he placed in my hands, but when he was not looking, I’d leave the coins
I’d managed to collect on the counter before I left. It was only a small change—if I had had
to buy a book with that pittance, I would probably have been able to afford only a booklet of
cigarette papers. When it was time for me to leave, I would do so dragging my feet, a weight
on my soul. If it had been up to me, I would have stayed there forever.

Page 10 of 53
One Christmas Sempere gave me the best gift I have ever received. It was an old volume,
read and experienced to the full. “Great Expectations, by Charles Dickens,” I read on the
cover. I was aware that Sempere knew a few authors who frequented his establishment and,
judging by the care with which he handled the volume, I thought perhaps that Mr. Dickens
was one of them. “A friend of yours?” “A lifelong friend. And from now on, he’s your friend
too.” That afternoon I took my new friend home, hidden under my clothes so that my father
wouldn’t see it. It was a rainy winter, with days as gray as lead, and I read Great Expectations
about nine times, partly because I had no other book at hand, partly because I did not think
there could be a better one in the whole world and I was beginning to suspect that Mr.
Dickens had written it just for me. Soon I was convinced that I didn’t want to do anything
else in life but learn to do what Mr. Dickens had done.

Q21) What is the main theme of the passage?

a) To show the authors' love for reading


b) To demonstrate the influential incident that made the author realize what he wanted
to do
c) To portray the relationship between the author and his father
d) Both (a) and (b)

Q22) What can be inferred about the father of the author from the passage?

a) He did not want the author to read


b) He lacked affection for the author
c) He was a stingy person
d) All of the above

Q23) What figure of speech is used in “with days as gray as lead,” as mentioned in the
passage?

a) Metaphor
b) Personification
c) Simile
d) Alliteration

Q24) Why was ‘Great Expectations’ the best gift received by the author?

Page 11 of 53
a) It was given to him by Sempere, who he valued very much
b) Nobody had ever gifted him a book before this.
c) It made him realize he wanted to be a writer
d) Because this book was better than any other he had read and he felt as if it was written
for him

SECTION 2 –GENERAL KNOWLEDGE

Directions (Q25-Q53): Read the following passages and answer the questions based on
it.

Passage 1 (Q25 –Q30):

The 15th Conference of Parties (COP) to the Convention on Biological Diversity (CBD)
ended on a high note with nearly 200 countries adopting a historic deal to preserve nature and
reverse damage to ecosystems after four years of fraught negotiations. India, which is one of
the Parties to the convention, pushed for Digital Sequence Information (DSI) to be
considered under the CBD's access and benefit sharing mechanism, among many other
targets during the run-up to the conference and through the anxious final moments until the
Kunming-Montreal Global Biodiversity Framework (GBF) became a reality.

Q25) COP 15 was held in _____?

(a) Montreal
(b) Kunming
(c) Glasgow
(d) Toronto

Q26) which of the following regarding the key targets of global biodiversity framework is
incorrect?

(a) Conserve and manage 30% areas (terrestrial, inland water, and coastal and marine) by
2030.
(b) Stop the extinction of known species, and by 2050 reduce tenfold the extinction risk
and rate of all species.
(c) Reduce global footprint of consumption by 2050.
(d) Reduce risk from pesticides by at least 50% by 2030.

Page 12 of 53
Q27) India enacted the Biological Diversity Act in which year to give effect to the provisions
of the convention on biological diversity?

(a) 2012
(b) 2004
(c) 2015
(d) 2002

Q28) Convention on Biological Diversity (CBD) has been ratified by how many countries?

(a) 193
(b) 196
(c) 189
(d) 187

Q29) When did the Convention on Biodiversity entered into force?

(a) 1993
(b) 1992
(c) 1995
(d) 1994

Q30) when was the environment ministry in India established?

(a) 1980
(b) 1988
(c) 1985
(d) 1990

Passage 2 (Q31 –Q36):

Guyana President [1] is set to be the Chief Guest at the 17th Pravasi Bharatiya Divas (PBD)
Convention, scheduled to be held in [2] in January next year. Pravasi Bharatiya Divas (PBD)
Convention is the flagship event of the Ministry of External Affairs and provides an
important platform to engage and connect with overseas Indians. Over 3,500 Diaspora
members from nearly 70 different countries have registered for the PBD Convention.
Addressing the gathering, the Prime Minister remarked that the Pravasi Bharatiya Divas is
taking place in all its glory after a gap of four years and highlighted the significance and joy
of personal interaction.

Page 13 of 53
Q31) why is Pravasi Bharatiya Diwas celebrated on 9th January?

(a) On this day in 1916 that Mahatma Gandhi, the greatest Pravasi, returned to India from
South Africa.
(b) On this day in 1909 that Mahatma Gandhi, the greatest Pravasi, returned to India from
South Africa.
(c) On this day in 1919 that Mahatma Gandhi, the greatest Pravasi, returned to India from
South Africa.
(d) On this day in 1915 that Mahatma Gandhi, the greatest Pravasi, returned to India from
South Africa.

Q32) theme for 2023 Pravasi Bhartiya diwas is _________

(a) Diaspora: Reliable partners for India’s progress in Amrit Kaal


(b) Role of Indian Diaspora in building a New India
(c) Engaging Diaspora: Connecting Across Generations
(d) Redefining engagement with the Indian Diaspora in Amrit Kaal

Q33) whose name is redacted with [1]?

(a) David Granger


(b) Irfaan Ali
(c) Donald Ramotar
(d) None of the above

Q34) Can OCI hold property in India?

(a) Yes, but they have to take permission from the central government.
(b) Yes, but they have to take permission from the respective state government.
(c) Yes, they can buy it without any permission.
(d) No, for buying any property in India one has to be a citizen of India.

Q35) When was the first Pravasi Bhartiya diwas was celebrated?

(a) 2000
(b) 2005
(c) 2003
(d) 2006

Q36) which city name has been redacted with [2]?

Page 14 of 53
(a) Bhopal
(b) Indore
(c) Gandhinagar
(d) Varanasi

Passage 3 (Q37-Q41)

The Golden Globe Awards will be held on January 10 (Jan 11 for India). This year, the
awards ceremony will stream LIVE in India on January 11, 2023. The Golden Globes will
felicitate the best in film and American television of 2022, chosen by the [1] .Comedian and
actor Jerrod Carmichael will be hosting the awards show. Ana de Armas, Jamie Lee Curtis,
Tracy Morgan are presenters for the ceremony.

Q37) which of the following was the first Asian song to win Golden Globe award?

(a) Jai Ho
(b) Naatu Naatu
(c) Take me to the sun
(d) The god wish

Q38) which film won the best motion picture drama award?

(a) Elvis
(b) Avatar: the way of water
(c) The Fabelmans
(d) Argentina,1985

Q39) whose name is redacted with [1]?

(a) Hollywood Foreign Picture Association


(b) Hollywood Films Press Association
(c) Hollywood Foreign Press Authority
(d) Hollywood Foreign Press Association

Q40) which edition of the Golden Globe awards was held in 2023?

(a) 81st
(b) 79th
(c) 80th
(d) 78th

Page 15 of 53
Q41) how many members are there in [1]?

(a) 100
(b) 190
(c) 105
(d) 110

Passage 4 (Q42-Q47)

[1] Was on Thursday sworn-in as Israel's prime minister for a sixth time, leading the Jewish
state's most right-wing government to date. Amid repeated heckling by opposing members
who called [1] "weak" and "racist", he told them to "respect the will of the voters," and that
"this is not the end of democracy or the end of the country.""In a democracy, you don't climb
over the walls of the Capitol," he said. He promised to improve the personal safety of the
citizens and lower the high cost of living.

Q42) whose name is redacted with [1]?

(a) Yair Lapid


(b) Benjamin Netanyahu
(c) Naftali Bennett
(d) Ehud Olmert

Q43) what is the parliment of Israel called?

(a) Knesset
(b) Folketing
(c) Riksdag
(d) None of the above

Q44) when did India establish diplomatic relation with Israel?

(a) 1950
(b) 1992
(c) 1998
(d) 1990

Q45) which of the following country(s) did not diplomatic relation with Israel?

1. Bahrain

Page 16 of 53
2. Saudi Arabia
3. Morocco
4. turkey
5. Egypt
(a) Only 3
(b) Only 1,2,3
(c) Only 2,4
(d) Only 1,3

Q46) which of the following is the capital of Israel as per India?

(a) Tel Aviv


(b) Jerusalem
(c) Both a and b
(d) Ramallah

Q47) Choose the incorrect statement(s)?

(a) The State of Israel is the only Jewish nation in the modern period
(b) Israel has developed from an agrarian state run along collectivist lines into a hi-tech
economy in the past 70 years
(c) The Palestinians in the West Bank and eastern Jerusalem have lived under Israeli
occupation since 1957.
(d) All of the above

Passage 5 (Q48-Q53)

In a talk on January 9, P. Sreekumar, the Satish Dhawan Professor at the Indian Space
Research Organization (ISRO) and advisor to its space science programmed, said that the
organization is yet to receive approval from the Indian government for the Venus mission and
that the mission could as a result be postponed to 2031.India's first interplanetary mission
'Mangalyaan' (Mars vehicle) was launched in 2013 and became the world's first-ever mission
to enter Martian orbit in the maiden attempt, back in 2014. Since then, there have been
mentions of India's follow-up Mars mission.

Q48) which of the following countries has sent a mission to Venus?

1. USA

Page 17 of 53
2. Japan
3. China
4. Russia
5. Europe
6. Canada
(a) Only 1,4
(b) Only 1,2,3,4,5
(c) Only 1,2,4
(d) Only 1,2,4,5

Q49) which of the following regarding Venus is incorrect?

(a) It is named after the Roman goddess of love and beauty. It is the second planet from
the Sun and fifth in the solar system in size and mass.
(b) It is the second brightest natural object in the night sky after the Moon.
(c) A day on Venus is longer than a year.
(d) All of the above

Q50) ISRO was established in which of the following year?

(a) 1965
(b) 1962
(c) 1969
(d) 1975

Q51) which of the following is not one of the missions that is to be launched to Venus?

(a) VERITAS
(b) EnVision
(c) Venusian
(d) DAVINCI

Q52) which of the following statement regarding Shukaryaan is incorrect?

(a) Its scientific payloads currently include a high-resolution Synthetic Aperture Radar
(SAR) and ground-penetrating radar.

Page 18 of 53
(b) The mission is expected to study Venus’s geological and volcanic activity, emissions
on the ground, wind speed, cloud cover, and other planetary characteristics from an
elliptical orbit.
(c) Shukrayaan-I will be launched on GSLV Mk I.
(d) All of the above

Q53) consider the following statements:

1. The Mangalyaan launched by ISRO is also called the Mars Orbiter Mission

2. Made India the second country to have a spacecraft orbit the Mars after USA

3. Made India the only country to be successful in making its spacecraft orbit the Mars in its
very first attempt

Which of the statement(s) given above is/are correct?

(a) 1 only
(b) 2 and 3 only
(c) 1 and 3 only
(d) 1, 2 and 3

SECTION 3 – LEGAL REASONING

Directions (Q54-Q83): Read the passages carefully and answer the questions.

Passage 1 (Q54 –Q58):

A contract of insurance is very similar to indemnity contracts. Here, the insurer promises to
compensate the insured for his losses. In return, he receives consideration in the form of a
premium. However, the Contract Act does not strictly govern these kinds of transactions.
This is because the Insurance Act and other such laws contain specific provisions for
insurance contracts. Parties under Indemnity Contracts
There are generally two parties in indemnity contracts. The person who promises to
indemnify for a loss is the Indemnifier. On the other hand, the person whose losses the
indemnifier promises to make good is the Indemnified. We can also refer to the Indemnified
party as the Indemnity Holder. For example, in the earlier example, C is the Indemnifier and
B is the Indemnity Holder. Nature of Indemnity Contracts An indemnity contract may be

Page 19 of 53
either expressed or implied. In other words, parties may expressly create such a contract as
per their own terms. The nature of circumstances may also create indemnity obligations
impliedly. For example, A does an act at the request of B. If B suffers some losses and A
offers to compensate him, they impliedly create an indemnity contract. Rights of an
Indemnity Holder When parties expressly make a contract of indemnity, they can determine
their own terms and conditions.
However, sometimes they may not do so. In such a case, the indemnity holder can enforce the
following rights against the indemnifier:
1) The indemnifier will have to pay damages which the indemnity holder will claim in a suit.
2) The indemnity holder can even compel the indemnifier to pay the costs he incurs in
litigating the suit.
3) If the parties agree to legally compromise the suit, the indemnifier has to pay the
compromise amount.

Q54) A wheelchair manufacturer enters into an agreement with a large hospital to provide
500 wheelchairs at a discount price. The manufacturer asks that a clause be included in the
contract, in which the hospital agrees to protect the company from any losses or lawsuits
should patients be injured while using any of the wheelchairs. Is the following an indemnity
clause?
(a) Yes, it is, as the patients would be compensated in case they face any injury.
(b) No, injured patients would be indemnified, and since they are not party to the contract
this won’t be an indemnity clause.
(c) Yes, as the hospital indemnifies the wheelchair company, or the hospital guarantees
for any losses or injuries that may occur.
(d) No, as the manufacturer is not protected from any litigations or compensatory
charges.

Q55) Lola has a homeowner’s insurance policy on her home in Texas. The insurance
company has agreed to indemnify Lola against damages to her home and the personal
property kept there, from many types of damage, including fire, burglary, and liability if
someone gets injured on Lola’s property. A visiting neighbor trips on a crack in the walkway
and falls, breaking her arm. Will her contract protect her from liability?
(a) No, as the indemnity is only in terms of injury from fire, burglary or someone getting
hurt due to these.

Page 20 of 53
(b) No, as the person tipped on the walkway which is not part of the home and outside
it’s periphery.
(c) Yes, the insurance company would protect Lola from the medical bills and other
losses claimed by the neighbor by paying the claim.
(d) Yes, as the contract of indemnity protects you from all the future losses incurred to
you.

Q56) Luke takes his car to the shop for repairs that will take a few days. The shop offers
Luke a loaner car so he can get back and forth to work. Luke signs the shop’s loaner car
agreement, which requires Luke to have insurance, and includes an indemnifying clause.
While driving through town in the loaner car, Luke rear-ends a car at a stoplight. Luke suffers
minor injuries, but the driver of the other car has several moderate injuries, and the damage to
the loaner car is substantial. A couple of weeks later, the other driver demands payment from
the repair shop, as owner of the car that hit her, for medical bills, repairs to her own car, and
pain and suffering. Who is responsible for this compensation?
(a) Luke as his written agreement to indemnify the repair shop protects the business
from liability.
(b) The lady will claim it from the repair shop and later they can make the deal with
Luke based on compromise.
(c) Luke, who is solely responsible for any events surrounding the accident, as he was
driving the car.
(d) The repair shop as they are the owner of the car and thus responsible for who they let
drive their vehicle.

Q57) Robert is remodeling his house to transform it into the home of his dreams. The front
porch and yard are being torn down and reconstructed, leaving Robert to worry about
whether one of the contractors may trip over debris and sue him for injuries. To protect
himself, Robert asks all of the contractors and laborers working on his home to sign an
indemnity and hold harmless agreement for the same. A worker while leaving the site for his
house, falls and injures himself over the pavement. Will he be indemnified by Robert?
(a) No, he won’t as the injury is not work related.
(b) Yes, he would, as he wouldn’t have been hurt if he wasn’t leaving from work itself.
(c) No, as the indemnity agreement protects him from paying for any liability caused due
to injury.

Page 21 of 53
(d) Yes, as he is the indemnifier for all liabilities caused to his workers employed on the
site.

Q58) Mr. X is a surgeon and currently doing practice in India. Mr. Y offers him Job in
Pakistan and asks him to start his practice here. Mr. Y also promises him that he will save
him from loss incurred due to practice here. Identify the indemnifier?
(a) Mr X is the indemnifier because he benefits from the contract.
(b) Mr. Y is the indemnifier because he would take up the liability.
(c) Indemnity contracts cannot work in foreign nations, as it is part of Indian Contract
Act.
(d) Third person would be indemnified as, indemnity contracts require a third person to
bear the liability.

Passage 2 (Q59 -Q83):

Exceptions to Offence of Murder under Section 300 IPC, which could mitigate it to culpable
homicide not amounting to murder are:-
Exception I – Grave and Sudden Provocation as mitigation Culpable homicide is not murder
if the offender, whilst deprived of the power of self-control by grave and sudden provocation,
causes the death of the person who gave the provocation or caused the death of any other
person by mistake or accident.
Essentials – The following conditions must be complied with in order to invoke the benefits
of this clause:-
1. The deceased must have given provocation to the accused.
2. The provocation must be grave.
3. The provocation must be sudden.
4. The offender, by reason of the said provocation, should have been deprived of his power of
self control.
5. The accused killed the deceased during the continuance of the deprivation of the power of
self control.
6. The offender must have caused the death of the person who gave the provocation or that of
any other person by mistake or accident.
Exception II – Exceeding the Right of Private Defence Culpable homicide is not murder if
the offender, in the exercise in good faith of the right of private defense of person or property,
exceeds the power given to him by law and causes the death of the person against whom he is

Page 22 of 53
exercising such a right of defense without premeditation, and without any intention of doing
more harm than is necessary for the purpose of such defense.
Exception III – Public servants exceeding his powers Culpable homicide is not murder if the
offender, being a public servant or aiding a public servant acting for the advancement of
public justice, exceeds the powers given to him by law, and causes death by doing an act
which he, in good faith, believes to be lawful and necessary for the due discharge of his duty
as such public servant and without ill-will towards the person whose death is caused.
Exception IV — Sudden Fight. This exception applies to cases wherein death is caused in a
sudden fight without premeditation in the heat of passion in a sudden quarrel; so long as the
fight is unpremeditated and sudden, the accused, irrespective of his conduct before the
quarrel, earns the mitigation provided for in Exception 4 to Section 300, IPC. Subject to the
condition that he did not in the course of fight take undue advantage of or act in a cruel or
unusual manner.
Exception V – Consent Culpable homicide is not murder when the person whose death is
caused, being above the age of eighteen years, suffers death or takes the risk of death with his
own free and voluntary consent.

Q59) Ram came home from work and found that his neighbor Sunil was molesting his 8
years old daughter. He was so shocked and angry that he took an ax and went towards Sunil
to kill him. Seeing him, Sunil pushed him and ran for his life. Ram ran behind Sunil with the
ax and found him 6 hours later hiding behind a bus. Ram caught him and killed him with his
ax. Decide whether the exception of Grave and sudden provocation will apply or not.
(a) Yes, because Sunil’s act could be considered as a heinous crime and was grave in
nature.
(b) Yes, because seeing his daughter getting molested, the father was filled with rage and
anger.
(c) No, because the act of killing Sunil was not sudden.
(d) Yes, because the act of Sunil deserves to be punished in this manner.

Q60) Raj was sleeping in his house when a thief entered his house. Due to the noise, Raj
woke up and caught the thief. The thief tried to threaten raj with a baseball bat kept there and
started to escape from the house. Raj in order to prevent the thief from escaping, took out his
pistol from the shelf and fired at him, thus killing him. Will he be sued for murder or any
exception can be granted.

Page 23 of 53
(a) No, because the thief was threatening raj and could have harmed it, therefore raj acted
in private defense.
(b) Yes, because killing is not a justified punishment for stealing.
(c) Yes, because Raj used more power than was necessarily required for private defense.
(d) No, because the thief committed a mistake and should have been punished.

Q61) X was a policeman who was trying to catch a dangerous armed culprit, who was hiding
in a factory. Unaware of this, the watchman of the factory who was taking rounds suddenly
appeared in front of the policeman holding his gun. As it was dark, “X” couldn’t see properly
and shot him dead. Will he be charged with murder?
(a) Yes, because it was because of his negligence that the watchman got killed.
(b) No, because he fired in the discharge of his duty, thinking him to be the armed
culprit.
(c) No, because it was an accident which took place because it was dark.
(d) Yes, because the policeman should have given warning before firing the bullet.

Q62) Mukesh and Nitin had a conflict due to which they had physically fought many times
before. Mukesh wanted to end this chapter so he called Nitin and asked to meet him in
person. He carried with him a knife. Soon their conversation got heated up and both got
involved in a physical fight. Finding the right time,Mukesh stabbed Nitin with his knife many
times. Will any of the exceptions apply here?
(a) Yes, the exception of grave and sudden provocation will be applicable as Mukesh
killed Nitin in the heat of the moment.
(b) No, because the exception of sudden fighting cannot be applied as it was planned by
Mukesh beforehand and was executed in a cruel manner.
(c) No, because he killed Nitin out of personal conflict and rage.
(d) Yes, because he acted in self defense against Nitin.

Q63) A 19 year old blind person named Jai was living with his guardian who used to be very
rude to him. He used to torture Jai over his disability and taunted him everyday that he is a
burden to everyone. Finally, Jai got depressed and requested his guardian to kill him in order
to give him relief from all the pain. Will the guardian be liable of murder?
(a) Yes, because the consent given by Jai was not voluntary and free will, he was forced
by his guardian.
(b) Yes, because he killed Jai because of his personal motive.

Page 24 of 53
(c) No, because the guardian relieved Jai from all the pain and sufferings.
(d) No because the guardian had the consent of Jai who was over the age of 18 and had
given voluntary consent.

Passage 3 (Q64 –Q68):

In an interesting order, Justice Rajiv Narain Raina of the Punjab and Haryana High Court
adjourned a Civil Revision on Petitioner pleading that the court was in a bad mood and he
would like to argue the case on some other day. The order dated February 4 discloses that
Justice Raina dismissed four urgent cases listed before him that day, one after the other. This
led to the Petitioner’s counsel, Advocate KS Sidhu, assuming that the court was not in the
right mood to allow his case. Counsel, assuming that the mood of the Court is bad this
morning dismissing the first four urgent cases one after the other with orders dictated in
Court, prays that time may be granted to him to argue the case on some other day. I grant
permission for an adjournment but not without saying that those cases were not worth
admission; the order read. The rules for adjournments are listed under Order XVII of the
CPC. Rule 1 thereunder clearly stipulates that the Court may grant an adjournment to a party
if sufficient cause; is shown provided that no such adjournment shall be granted more than
three times to a party during hearing. Sub- Rule (2) of Rule 1 further stipulates that
adjournments are to be granted only if the circumstances are beyond the control; of the party
who seeks it. However, these rules may be applicable only in cases of civil courts and not any
other authorities.

Q64) Anahad Narain was arguing before the NCDRC which is a commission defending
Piramal Healthcare Limited in a tough case. As the judge started grilling him on the nitty-
gritties of the matter, Narain started to find it uncomfortable as there were too many things he
did not have answers to. Narain feigns unconsciousness. As he pretended to regain his
consciousness, he sought an adjournment citing ill health from the Chairman of the
Commission. To his utter shock, the chairman denied such adjournment. Narain argued that
adjournment must be granted for circumstances beyond his control. Is the chairman bound by
Narain’s argument?
(a) Narain may be granted adjournment for he is not well in the mind of the judge.
(b) Narain may not be granted adjournment for he has feigned unconsciousness.
(c) Narain may be granted adjournment for the case is a tough one and he needs to
prepare.

Page 25 of 53
(d) Narain may not be granted adjournment for the CPC does not apply to Commissions
and thus they are not bound by the rule of circumstances being “beyond his control”.

Q65) Narain always feigns unconsciousness before a judge whenever he is put in a tight spot.
A case was being argued before the Calcutta High Court. Since Narain is now very
experienced at falling unconscious and seeking adjournments, he keeps doing it whenever he
finds a case difficult to deal with. As expected, This time too, the judge believes him to be
genuinely unwell.
(a) Narain may be granted adjournment for he is not well.
(b) Narain may not be granted adjournment for he has feigned unconsciousness.
(c) Narain may be granted adjournment for the case is a tough one and he needs to
prepare.
(d) Narain may not be granted adjournment for such acts against the interest of justice.

Q66) Narain is lazy and keeps procrastinating since the feigning adjournment business has
been working for him quite well now. Before the same judge, he took adjournments on
23/05/2016 and on 5/08/2016. The lawyer for the other side, learning from Narain’s tactics
also performs similar courtroom drama in the form of extreme headache and the judge grants
him adjournments on 29/08/2016 and 11/09/2016. Narain is again found to be in a tough spot
during the final arguments on 30/09/2016 and yet again seeks for adjournment. This time
however, Narain is genuinely feeling unwell because of too much partying last night.
(a) Narain may not be granted an adjournment for he’s the reason for such immense
delay.
(b) Narain may be granted an adjournment for circumstances beyond his control.
(c) Narain may not be granted an adjournment for the matter has been adjourned for more
than three times now.
(d) Narain may be granted an adjournment in the interest of justice.

Q67) Which of the following scenarios would be the most likely to be granted an
adjournment by the court?
(a) Devgan was arguing before Justice Amish of the Trivandrum Civil Court. He sought
an adjournment on the ground that his junior was sick and there was no one to take
care of the junior. Such circumstances were beyond his control and thus fell under the
relevant rule.

Page 26 of 53
(b) On 29/04/2019, having argued for almost 2 hours, Goswami stated that his medical
condition did not allow him to argue for more than 2 hours before the court and thus
sought for an adjournment. He has already sought adjournments on 23/01/2019,
24/02/2019 and 15/03/2019.
(c) Rabish submitted before the court that he wanted to pick his daughter up from school
and was thus constrained to seek an adjournment before the court.
(d) None of these would be eligible for an adjournment.

Q68) Laxmi, an advocate appears before the Karnataka High Court. While arguing, she seeks
her fourth adjournment on a frivolous ground. The judge however grants her an adjournment.
Decide
(a) The judge has acted illegally for he could only grant adjournments on reasons which are
beyond Laxmi’s control.
(b) The judge has acted illegally for he could only grant 3 adjournments to one person.
(c) The judge has acted legally because the rules above do not apply to his court.
(d) The judge has acted legally for adjournments at his discretion.

Passage 4 (Q69 –Q73):

The definition of private part has to be interpreted as per the social context, a Special Court
dealing with cases pertaining to the Protection of Children from Sexual Offences (POCSO)
Act said convicting a person for committing sexual assault under Sections 354, 354A of the
Indian Penal Code and Section 10 of the POCSO Act. (State of Maharashtra vs. Sahar Ali
Shaikh). The designated POCSO judge MA Baraliya had to consider whether the act of &
touching the bums &; of the victim by the accused would constitute an offense under Section
7 of the POCSO Act. Clarifying that “the term private part is to be interpreted into the
context of what is meant by it in our society,” the Court held that Google’s definition of not
including bum in the private part may not be an acceptable interpretation as far as Indians are
concerned.
Section 7- “whoever, with sexual intent touches the vagina, penis, anus or breast of the child
…or does any other act with sexual intent which involves physical contact without
penetration is said to commit sexual assault.” Accused and the other three were laughing at
her, when she went to buy bread. Second time when she was still going, they were laughing
at her. Past conduct of accused laughing at her and then touching her manifests that it was all
with sexual intention, to grab the chance. The Court noted that the accused had not touched

Page 27 of 53
the vagina, breast or anus of the victim. However, it ruled that touching the posterior of the
victim cannot be said to be without sexual intention. Sexual intention is the state of mind,
May not necessarily to be proved by direct evidence, such intention is to be inferred from the
attending circumstances of the case; the order read. While holding so, the Court sentenced the
accused to five years of rigorous imprisonment and payment of fine of Rs. 10,000.

Q69) Minakshi and Sheetal were best friends. One day they were playing in a park and
Minakshi saw a boy playing alone. She went to him and started playing with him.
Meanwhile, Suraj, father of the boy, saw Sheetal playing alone. He went to her and inquired
about her family. Taking the opportunity in favor, Suraj asked Sheetal to touch upon his
private body parts and offered her chocolates for doing so. He went on to make her feel
comfortable and grab her from behind and put his fingers across her private parts without
unclothing her. Minakshi saw the incident and went to Sheetal’s mother and stated the same.
An FIR was lodged against Suraj under POCSO. Decide-
(a) Suraj is liable for sexual assault of Sheetal as he had touched upon her private parts
and had undue physical contact with her, though he had no such intent to do so.
(b) Suraj is not liable for sexual assault of Sheetal as though he had touched upon her
private parts, he had not unclothed her and there was no skin-to-skin contact.
(c) Suraj is liable for sexual assault of Sheetal as he had sufficient intention to have
sexual contact with Sheetal and had touched upon her private parts is material.
(d) Suraj is not liable for sexual assault of Sheetal as though he had touched upon her
private parts, it’s not clear whether its vagina, breasts or anus and definition is still
unclear.

Q70) Payal was a young girl and pretty looking. She was the most beautiful girl in her
locality. She went shopping with her friend Saloni and was encountered with a gang of bad
boys who were stalking them for the past 10 minutes. She asked them not to follow them or
else she will call the police. Boys asked her to remain silent and let them enjoy her beauty.
They all were giggling and one of them commented that he wants to sleep one night with
Payal and another with Saloni. Payal got enraged and called up the police. The boy attacked
Payal and snatched her phone and ran away. Payal went to the police station and complained
of the gang. Decide-
(a) Sexual intention is to be inferred from the statements made by the boy and not only
by the physical acts or conduct. This can amount to sexual assault.

Page 28 of 53
(b) Sexual intention can only be concluded by direct evidence and has to be proved in
court of law. This cannot be said to be sexual assault.
(c) Sexual intention can be inferred from circumstantial evidences and not merely by the
act or conduct of the accused. But it alone cannot amount to sexual assault.
(d) Sexual intention has to be inferred from the conduct of the boys and even though
they have it, its a case of mere stalking and not sexual assault.

Q71) Mehul and Tamana were in a relationship for a long time. But their families were
against each other. They fixed Tamana’s marriage with Rahul to which Tamana agreed
readily. She asked Mehul to forget their long relationship and she cannot go against her
family decisions. Mehul got angry with Tamana and planned to take her from home and get
into sexual intercourse after which her family cannot deny their marriage. He asked Tamana
to meet her for the last time and take her to a hotel for dinner. He takes her to the bedroom
and makes her unconscious. He forcefully had sexual intercourse with her and asked her to
marry him. Tamana went to her family and stated the incident. They filed a case against
Mehul for sexual assault under IPC. The defence lawyer claimed that Mehul has not
committed alleged offence as he had used protection while having sexual intercourse and
there was no physical contact between them. Decide-
(a) The defence is correct as there was no skin to skin contact between Tamana and
Mehul and the act of sexual intercourse amount to sexual assault.
(b) The defense is incorrect as there was physical contact between Tamana and Mehul
irrespective of skin to skin contact and it amounts to sexual assault.
(c) The defence is correct as though there was sexual intention and forceful sexual
intercourse, the act has to be physical which not the case here is as Mehul used the
protection.
(d) The defence is not correct as the act of Mehul of forceful sexual intercourse with
Tamana amounts to much graver offence of rape along with sexual assault.

Q72) Shabana, a Muslim girl of 10 years old, was made through typical practices of her
religion from time long. She asked her mother about the same to which her mother responded
that Female Genital Mutilation and Nikah Halala were long set religious practices followed
by their community and thus everyone has to follow it. She claimed it to be wrong and
approached court to file complaint against Maulana who helped her family in conducting
FGM successfully. She alleged him of sexual assault as he had touched upon her private parts
without her due consent and should be punished for the same. Decide-

Page 29 of 53
(a) The court will held Maulana liable for sexual assault of Shabana as he had touched
upon her private parts and hence construed physical contact with her.
(b) The court will not held Maulana liable for sexual assault of Shabana as he was only
following the customary practices of their religion and had consented her family for
the same.
(c) The court will held Maulana liable for sexual assault of Shabana as there were enough
direct evidences to prove the alleged offence and religious practices is not an
exception to the offence.
(d) The court will not held Maulana liable for sexual assault of Shabana as there was no
sexual intention and physical contact with private parts were part of religious practice.

Q73) Section 354 of IPC and other related sections covered the offences of Sexual Assault
and Sexual Harassment prescribing the punishment with gravity of offence committed. Then
what is the need for implementing same clauses with different or graver punishments under
POCSO Act.
(a) Sexual Assault under IPC is different from POCSO and hence there was need to
redefine the same under the POCSO Act with higher punishment.
(b) Sexual Assault and Sexual Harassment offences are graver and heinous in cases of
child and to grant much protection to them, POCSO was implemented.
(c) POCSO Act is much more comprehensive in nature and gives detailed essentials of
Sexual Assault and Harassment then IPC and thus has overriding effect over it.
(d) IPC covers only limited number of cases of sexual assault and is gender neutral but
POCSO is not gender neutral and thus was need to implement it.

Passage 5 (Q74–Q78):

The incidence of income tax under the Income-tax Act, 1961 is based on the residence of the
taxpayer in a previous year, which is from 1st April to 31st March and the source(s) of his or
her income. While a person Resident; in India is taxed on his or her worldwide income,
nonresidents are only taxed on their India-sourced income. Section 6 of the ITA treats an
individual to be resident; in India in any previous year if he or she is in India for: (a) 182 days
or more in that year; or (b) 60 days or more in that year and has been in India for 365 days or
more in the four years preceding that year. A writ petition was filed before the Supreme
Court by a UAE-based NRI who, having come to India in March 2020 was finally able to
return to the UAE only after spending upwards of 182 days in India during FY 2020-21. The

Page 30 of 53
petitioner therefore sought a direction to the effect that he would be considered non-resident;
for Indian income tax purposes for FY 2020-21, irrespective of the duration of his stay in
India, on account of the COVID-19 pandemic. Taking note of the fact that certain relaxations
had been granted by the Central Board of Direct Tax for FY 2019-20 through the Circular,
and the fact that the pandemic had continued beyond March 31, 2020, because of which
many people had remained stranded in the country, the Supreme Court directed the petitioner
to make a representation to the CBDT and further directed the CBDT to consider the same
within three weeks of the receipt thereof. English courts have held since time immemorial
that residence must be adopted voluntarily and for settled purpose, and Indian courts have
followed suit. For instance, in CIT v. Suresh Nanda, the Delhi High Court held that where a
person was compelled to stay in India because his passport was impounded had to be
excluded in determining whether he was resident; in India for the relevant assessment year.
Delhi high court held that if the record discloses materials that the stay (to qualify as resident
Indian) lacked volition and was compelled by external circumstances beyond the individual’s
control, she or he could not be treated as a resident Indian.

Q74) Dheer was an Indian citizen, who worked at TCS, Bengaluru as a consultant for a
salary of INR 55,000 per month. He worked there from 01 April 2019 to 27 September 2019.
After he resigned, he applied for jobs at multinational companies in foreign countries for
better career prospects. On 01 October 2019 he left for the USA, where he had been invited
to work as chief project engineer for SpaceX for a salary of INR 1,20,000. He worked there
from 1.10.2019 to 31.3.2020. His salary for October to December 2019 was credited in his
USA bank account and the salary for January to March 2020 was credited in his Bombay
account directly. Decide which of his income would be taxable in India for the previous year
2019-20.
(a) Since he is a citizen of India, all the income that he earns anywhere around the
world would be taxable in India as per the Income Tax Act.
(b) Since he is a non-resident only the income that he earned from TCS in the
concerned previous year would be taxable in India as per the Income Tax Act.
(c) Since he is a resident, all the income that he earns anywhere around the world
would be taxable in India as per the Income Tax Act.
(d) Regardless of his residential status, the income earned from TCS and the income
from Google that was deposited in a Bank account in India would be taxable in
India as per Income Tax Act.

Page 31 of 53
Q75) Which of the following is/are not a conclusion of the article:
A. Only Indian residents are liable to pay tax on their worldwide income, whereas non-
residents have to pay taxes on only income earned in India.
B. A person shall be liable to pay taxes if they satisfy the mere letter of the provisions and
voluntariness of their stay is irrelevant in the determination of their residential status.
C. Subject to factual circumstances, a person who was compelled to stay for external reasons
shall not be liable to pay taxes as a resident of the nation.
D. The Supreme Court upheld the ruling of Delhi High Court and allowed the petition by
excluding the period of involuntary stay of the petitioner in India for the purpose of
determining residential status.
Choose the correct answer:
(a) II and IV
(b) I and III
(c) I, III and IV
(d) III and IV

Q76) Azaan was a spice trader from Afghanistan, who imports spices from India. Once,
when he was in to close a deal, Pakistan declared a war against India. As a result, all of the
Indian borders were sealed and no person was allowed to enter or leave the country. The war
lasted for 7 months and for all this time Azaan was compelled to stay in India. After the war
ended, he prepared to go back to Afghanistan, however, he received a notice to pay income
tax on all the income he had earned in the previous year as he qualifies as a resident of India.
He has contended that he was forced to stay in the country due to the ongoing war and did not
volunteer to do so. The case is now pending before a high court. Which of the following is
the correct answer?
(a) Azaan shall be liable to pay income tax on his entire income earned worldwide as he
has satisfied the criteria of being an Indian resident.
(b) Azaan shall be liable to pay income tax on his entire income earned worldwide as the
source of all his income is the spice that he imports from India
(c) Azaan shall be liable to pay tax on only income earned in India as he is an Afghan
citizen and the Income Tax applies on only Indian citizens.
(d) Azaan shall be liable to pay income tax on only income earned in India as he has been
involuntarily staying in India and could not return to Afghanistan due to the war.

Page 32 of 53
Q77) Mitchell was a lawyer from the UK, who was in India to represent a multinational
automobile company. When the Supreme Court reserved the judgment to be given after 3
months, the lawyer decided to stay to spare the cost of traveling. The judgment was given in
the favor of the multinational company Mitchell prepared for going back to the UK. When he
filed returns to pay tax on income earned in India, he was notified that he had stayed in India
for a period of 193 days and thus was an Indian resident for the previous year, and thus shall
be liable to pay tax on all the income earned. He appealed against the notification before the
Income Tax Appellate Tribunal on the grounds that he was compelled to stay in India
because the judgment was reserved for a later date. Choose the correct option.
a) Appeal shall be allowed as Mitchell did not volunteer to stay in India and was under
compulsion to save the cost of traveling, thus the period of his stay shall not be
counted to determine his residential status.
b) Appeal shall not be allowed as Mitchell was under no compulsion to stay in India and
volunteered to stay for a specific purpose and thus is a resident of India
c) Appeal shall be allowed as Mitchell is just representing the company in India and
payment for that case would not be income earned in India Therefore, Mitchell shall
not pay any taxes on his income in India
d) Appeal shall not be allowed as payment received to defend a client in India amounts
to income in India which is taxable regardless of the residential status of Mitchell.

Q78) Anas was a student from Bangladesh, completing his bachelors and masters degree
from India in 2015. In November 2021, he decided to go back to Bangladesh to start up a
tech firm of his own. His firm did very well in the very first year and he earned enough to pay
off all his education loans. In April 2022, he received a notice from the Assessing Officer of
Income Tax Department, Delhi to pay tax on all the income he earned in the previous year
2021-22. He has made representation before the authority and contended that he did not earn
a single penny in India and all his income in the relevant year has been earned in Bangladesh
only. Which of the following is the correct option?
(a) Anas shall be liable to pay income tax on all the income he earned including that in
Bangladesh as in the relevant previous year, he was a resident of India
(b) Anas shall not be liable to pay any taxes in India for the relevant previous year as he
did not earn any income in India even if he was a resident in that previous year.
(c) Anas shall be liable to pay income tax on only the income that has been earned in
India, if any, in the relevant previous year.

Page 33 of 53
(d) Anas shall not be liable to pay any taxes in India as he is a non-resident and he did not
earn any income in India in the concerned previous year.

Passage 6 (Q79–Q83):

Every person who enters into a profession assures of a skilled and efficient work. He/she
assures to work with reasonable care and skill and guarantees that the skills possessed by
him/her shall be exercised with due diligence. Similarly, a medical professional does not
guarantee the results of treatment to a patient. A surgeon may not assure of a 100%
affirmative result of the operation but guarantees that the skills possessed by him in that
branch of practice would be performed by him/her with reasonable competence.Therefore,
according to these standards, a professional would be negligent under two instances-
first,either the professional did not possess the requisite skills he/she was required to possess;
secondly, the professional did not exercise with reasonable competence. Any person who
provides medical treatment or advice to others impliedly undertakes his possession of
knowledge and skills. Therefore, such a person has certain duties to consider, such as
decision regarding the undertaking of a case by him/her and decision regarding the kind of
treatment to be administered. Any breach of such duties indicates negligence on the person’s
part and gives a right of action against him/her. As it is said, to err is too human, but causing
injury to others due to mere carelessness is negligence. However, an error in judgment by a
medical professional does not amount to negligence as due to change of circumstances or
even in the same circumstances, there can be instances where the decision of the doctors goes
wrong. But, considering the essential factors related to treatment, if a wrong judgment is
taken, then it would be negligence as the medical professional had the efficient knowledge of
the factors. The Supreme Court of India in Kusum Sharma vs. Batra Hospital held that a
doctor adopts a procedure involving a higher element of risk, but he/she has an honest belief
of greater chances of success in the treatment. Therefore, if the doctor performs the treatment
and it does not yield affirmative results, then it does not amount to medical negligence.

Q79) Mr. Purohit was a reputed cardiologist in Gupta Hospital and runs a private clinic in
another hospital. Shushma, a 60 year old diabetic woman, had breathing problems. She was
not able to find a good doctor to cure her disease. Her friend, Ms. Leela suggested that Dr.
Purohit asked her for consultation. Shushma took his appointment and Mr. Purohit asked her
to undergo typical tests to confirm whether she is suffering from asthma or not. Unaware of

Page 34 of 53
Shushma being claustrophobic, she was exposed to typical tests and as a result of which she
died. Decide –
(a) Mr. Purohit cannot be held liable for this negligence as such facts regarding personal
information have to be conveyed by Shushma voluntarily before the tests.
(b) Mr. Purohit and the hospital can be held liable for negligence as it is his duty to
collect relevant information of the patient before putting them to tests.
(c) Mr. Purohit cannot be held liable as he was not negligent. It is not his duty to ask
Shushma regarding her personal information.
(d) Mr. Purohit can be held liable because he should have applied his due diligence to
collect relevant information from Shushma before putting her to tests.

Q80) Aryan was a special child as he was born with certain vitamin deficiencies and had
undeveloped bone marrow. His parents consulted various doctors and therapists and
approached Dr. Malti to cure their child.Doctor recommended several tests including blood
pressure and sugar checkups. After going by the reports, Dr. Malti prescribed certain heavy
dose medicines to the child and asked the parents to revisit after a month to observe the effect
and curing of the child. After 15 days of such an appointment, Parents approached Dr. Malti
and alleged of her incompetency. Dr. Malti was unable to understand the situation and came
to know that she misread the count for white blood platelets as red blood platelets and had
given wrong tablets. She is –
(a) Not liable for medical negligence as she had only misread the reports and cannot be
alleged for not taking due care and diligence for prescribing medicines.
(b) Liable for gross medical negligence as she was professionally competent and had
efficient knowledge of the effect of prescribing wrong medicines to the child.
(c) Not liable for medical negligence as she was only at fault for misreading the reports
and can be said to have taken the wrong decision but not negligent.
(d) Liable for medical negligence as she should take reasonable care and use her due
diligence before going by the reports as she was a professionally competent doctor.

Q81) Rahul was a 4th year student of M.B.B.S. Before his graduation, he went for a short trip
with his friends. On his trip, he encountered new experiences and explored different sides of
his studies. On one fine day of the trip, Rahul encountered an old lady, badly wounded, lying
on the path and was asking for help. Rahul analyzed the problem and suggested she apply
paste of wild plants to cure the wound. He himself made the paste and applied to the wound.

Page 35 of 53
Due to the poisonous content in the wild plant, it got mixed with the blood and eventually the
lady died. Rahul can be-
(a) Held liable for medical negligence as he was not having proper knowledge regarding
the components of wild plants and had not applied reasonable care before applying it
to the wound.
(b) Held not liable for medical negligence as though he is professionally competent but
does not have requisite skills and knowledge regarding components of wild plants and
was helping the lady to cure the wound.
(c) Held liable for medical negligence as he was completing his graduation and can be
implied that he possesses sufficient knowledge and skills of performing such medical
help to the injured persons.
(d) Held not liable for gross medical negligence as he is not professionally competent to
perform such medical help to injured persons and he does not possess such requisite
skills or knowledge for the same.

Q82) Abhinav was a brilliant student and has secured a good rank in his college times. He
studied the other fields of biology and Vedic science and has researched much upon these
aspects. Soon he opened his own Hamdard Dawakhana and relied on homeopathic curing of
diseases. Nirmala listened to much of this Dawakhana and visited for treatment of her
pancreatic pain. Due to overcrowding and rush at Dawakhana.
Abhinav gave Nirmala wrong pills and as a result her internal reproductive parts got
damaged. She claimed compensation from Abhinav for her treatment. Abhinav refused and
contended that it was a mere mistake and he is not liable for negligence. Decide-
(a) Abhinav is not liable for medical negligence as he rightly contended that it was mere
mistake on his part and not negligence in prescribing wrong medicines to Nirmala.
(b) Abhinav can be held liable for negligence as he should take reasonable care and apply
his due diligence before prescribing medicines to Nirmala.
(c) Abhinav cannot be held liable for medical negligence as he is not professionally
competent and does not have requisite skills or knowledge as to cure such diseases.
(d) Abhinav can be held liable for medical negligence as he has implied knowledge or
skills as to the effect of such homeopathic medicines and should pay compensation to
Nirmala.

Q83) Shruti, a pregnant lady, was having her child after 20 years of her marriage. She was
under continuous guidance of a gynecologist, Dr. Suman. She analyzed the critical

Page 36 of 53
circumstances and stated that there can be complications in delivering the child of Shruti as
she was diabetic and hence to undergo surgery. Shruti insisted on normal delivery but Dr.
Suman refused the proposal. Delivery was done perfectly but while doing the surgery, the
Suman forgot to stitch an organ properly and it was left unclosed. Since no harm was caused
by Shruti, Dr. Suman asked her to perform another surgery to mend the mistake. Shruti can –
(a) Held Dr. Suman is liable for medical negligence as she should have taken reasonable
care while performing the surgery and used her due diligence while stitching the
parts.
(b) Held Dr. Suman was not liable for medical negligence as she was not professionally
competent to perform the surgery and had done a mistake on her part.
(c) Held Dr. Suman is liable for medical negligence as Shruti asked for normal delivery
but she has refused for it and is liable for her decision to undergo surgery for
delivering the child.
(d) Held Dr. Suman was not liable for medical negligence as she was correct with her
decision to perform surgery and not normal delivery of the child and had committed a
mere mistake while performing surgery.

SECTION 4 - LOGICAL REASONING

Directions (Q84-Q108): Read the passages carefully and answer the questions.

Passage 1 (Q84 –Q88):

Boasting wingspans of up to 11 feet-largest of any bird alive today-these feathered goliaths,


native to the Southern Ocean and North Pacific, are built to soar. Gliding at speeds exceeding
50 miles per hour, they can cover vast swaths of sea in minutes, whilst scouring waters for
bright flickers of fish. Some species are known to spend years at sea without touching down
on land, and a few have even been documented circumnavigating the globe. With their keen
eyes and wandering ways, albatrosses are the de facto "sentinels of the sea," says Henri
Weimerskirch, a marine ornithologist at French National Center for Scientific Research.
Weimerskirch is working to make that title a little more official-by recruiting seabirds to
patrol oceans for illegal fishing vessels. He and his colleagues have outfitted nearly 200
albatrosses with tiny GPS trackers that detect radar emissions from suspicious ships, allowing
the birds to transmit locations of fishers amid illicit acts. Results of this method were
published in a study in the Proceedings of the National Academy of Sciences. Capable of

Page 37 of 53
following fishing boats into remote regions out of reach of monitoring machines like ships,
aircraft and even individual satellites, these feathered crimefighters could offer a convenient
and cost-effective way to keep tabs on foul play at sea-and may even help gather crucial
conservation data along the way.
"This is a clever method for facilitating law enforcement," says Melinda Conners, a
conservation biologist studying albatrosses at Stony Brook University who was not involved
in the study. "There is no boat or plane that can match the capability of an albatross to cover
vast oceanic regions." In many parts of the world, trawling for marine catch has become
something of a fishy business. Though estimates vary, around a fifth of fishes on the market
may be the result of illegal, unreported and unregulated catches at sea. Fishers who trawl
without a license, exceed quotas or underreport their hauls imperil fragile ecosystems, and
their operations have been linked to other crimes, including human trafficking. All told,
illegal fishing is thought to cost the global economy up to $20 billion or $30 billion every
year-figures that exceed the annual GDP of about half the world's countries. However,
policing the ocean, which blankets more than 70 percent of our planet, is no easy task for
land-based law enforcement agencies. Resources and infrastructure are notably lacking far
from the coast, where regulated strips of the ocean give way to international waters, says
Amanda Gladics, a fisheries expert at Oregon State University, not involved in the study. Out
here, surveillance often relies on something of an honor system, wherein vessels voluntarily
report their presence through an automatic identification system (AIS) that can easily be
switched off. "If any boats cut off its AIS, nobody knows where the boat is," Weimerskirch
says.

Q84) It can be inferred from the passage that:


a) Albatrosses can detect vessels that have their automatic identification systems
switched off.
b) Satellite tracking systems cannot reach the remote swaths of the high international
waters.
c) Surveillance and policing by albatrosses extend to over 70% of the oceans on the
planet.
d) Albatrosses are more effective than enforcement agencies in keeping a tab on illicit
fishing.

Page 38 of 53
Q85) All of the following are true about the surveillance carried out using an albatross
EXCEPT:
a) Certain officials have actualized the project of employing Albatrosses in surveillance.
b) Albatrosses have been trained to send back signals through GPS trackers fitted on
them.
c) An Albatross will follow illegal fishing ships in far off regions that are hard for
humans to track.
d) The use of Albatrosses for surveillance purposes does not appear to be a very
expensive venture.

Q86) According to the passage, which of the following can be inferred from the passage
above
a) No other living bird is as large as the Albatross.
b) Albatrosses are native to colder areas of the polar region.
c) Albatrosses fitted with GPS trackers help check human trafficking
d) Albatrosses can fly long distances, even around the world.

Q87) Which of the following, if true, would most strongly invalidate Weimerskirch's project
(of recruiting Albatrosses to patrol the ocean for illegal fishing vessels)?
a) Albatrosses are naturally attracted to fishing vessels because these vessels are a
source of easy food for the Albatross.
b) Vessels constantly emit radar for the purpose of navigation and to avoid collisions
with other marine bodies.
c) The range of radar signals from vessels is not big enough to be picked up by
Albatrosses unless they get close enough to the vessels.
d) The devices attached to the plumage change the birds' natural routes and they tend to
avoid long flights.

Q88) Which of the following statements cannot be inferred according to the passage?
a) Albatrosses are fitted with GPS-enabled devices that send back signals about
suspicious ships.
b) the GPS tracker can detect the ship whose AIS has been turned off
c) It is hoped that ships involved in criminal activities will not become aware of the
purpose of Albatrosses used in surveillance.

Page 39 of 53
d) The GPS-enabled devices fitted on Albatrosses can track the radar emissions from
ships.

Passage 2 (Q89 –Q93):

In the essay, "The Transaction", William Zinsser writes about a doctor who has recently
begun to write and has experienced some publishing successes. He compares his way of
working with
the way the doctor works. Zinsser points out that to him, a professional writer, writing is a
vocation, while to the doctor, it is an avocation. I always wanted to call writing my vocation.
Like many people, I had a lifelong dream of being a writer. I began to teach. I never viewed
teaching as my vocation. First and foremost, I was a writer. The teaching was just something
I did-a class or two a semester-on the side. Only "on the side" took up a huge portion of what
I'd anticipated would be my writing time. I am conscientious and hardworking by nature, and
approach everything I take on with gusto. Teaching was no different. I was dedicated to
helping my students discover and develop their individual voices. However, I decided to take
a break the following spring to put into practice the subject I'd been teaching and pursue my
vocation. I was going to write. What happened that spring is as unsurprising as a predictable
plotline. With time stretched out endlessly before me, I filled it just as endlessly with writing-
related activities, all of which provided a pretext of writing but produced little new work. I
am an impulsive person and one day, five months into my vocation as a full-time writer, I
picked up the newspaper, studied the wanted ads and started to send out my resume. I quickly
progressed from applying for part-time to applying for full-time positions, reasoning in my
non-writing angst that as long as I was going to compromise on my dream and work for
someone else, I might as well be well paid for my efforts. When I began my new position as a
contract administrator for a real estate broker, I didn't know what to expect. So, what did I
discover? It was a refreshing change to work hard and have not only my boss, but all his
clients tell me what a great job I was doing. My communication skills, both oral and written,
and the requirements of the job were a perfect match. I'd found my vocation. Maybe the
ending is to be expected, a plot twist in what continues to be a predictable storyline. I still
write. Now that writing has become my avocation, I have become more prolific despite, or
maybe because of, having to squeeze my writing into narrow periods of time.

Q89) The author would describe avocation as a:


a) Full time job

Page 40 of 53
b) Part time job
c) A hobby
d) Life's Pursuit

Q90) Which of the following is the most likely reason for the author not being able to fulfill
her writing dreams?
a) She was not ready to take the risk of being a full time author.
b) She was not good enough.
c) She was not passionate enough.
d) She was never meant to be a writer; it was only an infatuation.

Q91) Which of the following can be concluded from the above passage?
a) Writing requires as much effort and dedication as any other vocation.
b) Education in the literary arts is a waste of time.
c) Writing can never be your full time job.
d) Writing cannot help sustain you financially.

Q92) Consider the statement, "Maybe the ending is to be expected, a plot twist in what
continues to be a predictable storyline." The twist in the plot refers to:
a) Resumption of writing by the author, but in a different scenario.
b) The author is taking up a full time job.
c) The author enjoying her new role and being successful at it.
d) The author finally realizing the futility of her dream and giving up

Q93) Strengthen the following statement “I never viewed teaching as my vocation. First and
foremost, I was a writer. The teaching was just something I did-a class or two a semester-on
the side.”
a) Teaching as a profession is more enjoyable than being a full time author.

b) Teaching is something the author seeks as a long term goal in life.

c) The author is focused on writing and enjoys that even though it is now not his main
job.

d) The doctor who turned to writing books was following his passion.

Passage 3 (Q94 –Q98):

Page 41 of 53
Fifty years ago, only New Yorkers lived in what is now termed a “megacity,” an
agglomeration of more than 10 million people living and working in an urban environment.
In contrast, today there are more than 40 megacities, most in less developed countries, and
more urban centers are expected to explode in population by the year 2020. Demographers
and globalization experts are already referring to the 21st century as “the urban century.”
Already, more people on our Earth live in cities than live in rural areas. This is an enormous
change in population trends, and it skews the entire planet in ways we haven’t begun to
analyze. Although some cities have seen immigration expand their borders, for most
megacities, it is migration from within the country that has caused the city to grow. An
example is China, where some 150 million rural inhabitants have migrated to cities in just the
last ten years. In many cases, the cities house the only possibilities of employment in this
global economy. That is what has grown Mumbai (Bombay), India, from a large city to a
megacity of more than 18 million people in just a few years. It’s the cause of the explosion of
populations in Lagos, Nigeria; Karachi, Pakistan; Dhaka, Bangladesh; and Jakarta, Indonesia.
Whereas just a few years ago, most large cities were in developed nations, now the largest are
suddenly in the less-developed countries of South America, Africa, and Asia. Imagine the
pressure on the infrastructure of these already poor cities as the influx of workers pushes
services to the breaking point. Slums and shanty towns spring up around the outskirts of the
cities, and the government is powerless to affect the disadvantaged workers, leaving them
exposed to corrupt local officials or urban gangs. Imagine, too, what happens in the rural
areas that these people have abandoned. China faces a desperate shortage of agricultural
labor. So do other areas of Asia and Africa. According to UN statistics, by the year 2030,
more than 60 percent of the world population will be urban, up from 30 percent in 1950.
Unlike the population growth in developed nations, the birth rate in less-developed nations is
high, meaning that the cities continue to grow even as migration slows from the rural areas.
Megacities such as New York have populations that have leveled off over time. Despite its
location in a less-developed nation, even a megacity such as Mexico City has a slow rate of
growth compared to Asian and African cities such as Mumbai or Lagos. It is difficult to
imagine what the growth of the megacities will mean to the world in the 21st century.
Demographers foresee ecological overload, homelessness, uncontrolled traffic, and an
infrastructure strained to the breaking point. Despite the notion that industrial jobs improve
the lot of the workers, it is already possible to see that megacities are creating a new, even
deeper division between rich and poor, as the poor concentrate in the outskirts of town and
the rich barricade themselves behind walls and in towers.

Page 42 of 53
Q94) The main argument of the passage is that:
a) megacities are more often found in less-developed nations but strain the resources of
developed nations.
b) the growth in population and number of megacities means foreseeable changes,
many of them negative.
c) The movement of population bases from rural to urban locations decimates the
countryside and limits our ability to grow food.
d) we must begin to fight back against the growth of megacities in the less developed
nations of the world.

Q95) The passage suggests that demographers:


a) have not been able to keep pace with the growth of cities.
b) focus primarily on population trends in the developing world.
c) are observing the growth of the world’s cities with concern.
d) work hand in hand with the UN to plan for the future.

Q96) The author’s use of UN statistics helps:


a) strengthen her argument that urbanization is radically changing the world.
b) contradict demographers’ claims about megacities and their effects.
c) indicate that the results of urbanization include poverty and crime.
d) complement her assertion that birth rate is the main reason for urban growth.

Q97) According to this passage, why might skyscrapers be a sign of divisiveness?


a) They cost too much to build.
b) They are found only in developed nations.
c) They separate the rich from the poor.
d) They house businesses, not people.

Q98) Which new information, if true, might CHALLENGE the author’s contention that cities
will continue to grow despite a slowing of migration from the countryside?
a) Scientists are creating new strains of rice and wheat that require far less in the way of
hands-on care.
b) The number of people living below the poverty level will climbin less-developed and
developed nations.

Page 43 of 53
c) Inflationary trends in heating oil and gasoline prices will limit most people’s
discretionary spending.
d) New methods of birth control will limit the population explosion in the developing
world.

Passage 4 (Q99 –Q103):

In the build-up to the 'Leaders' Climate Summit' organized by the United States this week,
there has been a flurry of articles about whether India should announce a 'net-zero' emissions
target, and by when. The Intergovernmental Panel on Climate Change (IPCC) 1.5°C report
called for global carbon emissions to reach net-zero by 2050, which the pressure cooker of
climate diplomacy has quickly transformed into a call for all countries to announce 2050 as
the net-zero target year. Yet, global net zero may require some countries reaching net-zero
before 2050 in order for others to have some additional time. Since a disproportionate share
of the carbon space has been used up by developed countries, it is important that they act
boldly at home, to match the vigor of their diplomatic efforts.
Nonetheless, as a climate-vulnerable country, India must also up its game to contribute to
limiting global temperature rise, ideally below 1.5°C. While doing so, it should not lose sight
of the history of global climate negotiations and its own developmental needs. Though a large
country and economy, we are still a very poor country with a significant development deficit
- for example, our per-capita carbon emissions are less than half the world average. So, what
is the way forward for India? Saying India will take only modest steps until richer countries
do more is not viable in the context of a global climate crisis. Yet, announcing an Indian 2050
net-zero commitment risks taking on a much heavier burden of decarbonisation than many
wealthier countries, and could seriously compromise India's development needs. We suggest
a third path, focused on concrete, near-term sectoral transformations through aggressive
adoption of technologies that are within our reach, and an earnest effort to avoid high carbon
lock-ins. This is best accomplished by focusing on sectoral low-carbon development
pathways that combine competitiveness, job-creation, distributional justice and low pollution
in key areas where India is already changing rapidly. This approach is directionally consistent
with India moving net-zero, which should be our long-term objective. Here, we detail what
such an approach would look like, by laying out the contours of an enhanced national pledge
for the electricity sector, to illustrate how it can be both ambitious and in India's interest. To
achieve net-zero emissions, a key piece of the puzzle is to decarbonise the electricity sector,
which is the single largest source (about 40%) of India's greenhouse gas emissions.

Page 44 of 53
Decarbonised electricity would also allow India to undertake transformational changes in
urbanization and industrial development, for example by expanding the use of electricity for
transport, and by integrating electric systems into urban planning. India now needs to shift
gears to a comprehensive re-imagination of electricity and its role in our economy and
society.

Q99) Which of the following would be the most suitable title for the given passage?
a) Urbanization and industrial development through decarbonized electricity.
b) India's net-zero target commitment
c) A low-carbon future for India via sector-led change.
d) India's and the world's climate crises.

Q100) According to the given passage, which of the following strengthens the author’s
conclusion
a) India's would have a great advantage over their economy if they diplomatically shift
the commitments cutoff date to 2075
b) India's per-capita carbon emissions are more than half the world average.
c) About 96% of India’s power generation is dependent upon coal
d) 2050 net zero commitment should first be honored by the first world countries-
mainly of Europe and America.

Q101) What, according to the given passage, could happen to India if it announces 2050 net-
zero commitment?
a) It risks taking on a heavy burden of decarbonization, more than many wealthier
countries and could compromise its developmental needs.
b) It risks taking on a heavy burden of decarbonization, more than other developing
nations that are in a similar predicament.
c) It could be caught unprepared to meet such a commitment that might result in
economic and political fallouts.
d) It could leave the wealthier nations less responsible about their 2050 net-zero
commitments.

Q102) What is the author’s conclusion for the passage given above?
a) The time is not right yet for India to bring about net zero carbon emissions

Page 45 of 53
b) India must focus on concrete, near-term sectoral transformations through aggressive
adoption of technologies that are within our reach, and an earnest effort to avoid high
carbon lock-ins.

c) India is an extremely climate vulnerable country.

d) India is a very poor country with a significant development deficit and our per-capita
carbon emissions are less than half the world average.

Q103) Which one of these is a valid assumption made by the author?

a) A lot of the developments made by India in the prior centuries would be decimated if
the current leaders are not careful about the way they approach the current climate
negotiations.

b) The world treaties made in furtherance of climate development do not yield any
substantial changes in the long run.

c) The world leaders are strategically pursing for the development of their industrial
landscape while the environmental concerns take a backseat.

d) None of the above

Passage 5 (Q104 –Q108):

The world has entered the new Axial Age. Numerous transformations are taking place in the
models of social, economic, and political activity, in projections of power and authority. The
political landscape and its relevant "content structures" like democracy and liberalism, right
and left, globalization and nationalism etc, are acquiring new systemic qualities. If we want
to avoid fighting with the ghosts of the past, it is necessary not only to take into account these
transformations but examine them from within. To see the complexity of things, to
understand the transformation of the world in transit, we need to get rid of myopic, linear
interpretations of seemingly familiar but morphing notions like "globalism and nationalism".
Who can manage the complexity of the "plurilateral" world we are stepping into? What kind
of political architecture will be needed to support the nascent multi- or rather "pluripolarity"?
How can we synchronize governance with inevitable digitalisation of politics?
How can we enable decision-making mechanisms at the global level? These questions need
to be answered. Nobody will miss the train to "a bright tomorrow". Historical time flows for
everyone-you cannot hide "behind the wall" to avoid it. No actor of a historical process can
bury his head in the sand of the comfortable present, indulging in "counter-clockwise revolt".
The future will come for everyone, but not everyone will hold an equal place in it.

Page 46 of 53
Q104) Based on the arguments given by the author, which of the following would be the best
approach in understanding the growing level of nationalism globally?
(a) Comparing the attributes of nationalism with those of globalism in an analytical way
(b) Shunning simplified interpretations by not comparing one global phenomenon with
another
(c) Analyzing the systemic qualities that have been newly acquired by some
transformational phenomenon
(d) Sorting the relevant content structure for the issues that need immediate attention

Q105) Which of the following problem-solution pairs, who are most closely related to the
"ghosts of the past" section of the passage?
(a) The habit of hygiene, if not developed in childhood, would never become a part of an
individual's behavior.
(b) People who have faced trauma in their past need to be in the presence of happy people
for a prolonged period of time.
(c) The exaggeration of one's troubling days by successful people makes them appear
more successful than they are.
(d) The struggles of poor living standards of people could only be understood by realizing
the details of their everyday life.

Q106) Which of the following best encapsulates the notion that the author is trying to convey
by means of the multiple questions that are mentioned in the passage?
(a) Fear of tackling and doubts on leaders in their ability to handle the collective
challenges that the future has in store for those leaders
(b) Lack of will to transfer power and responsibilities to others who could face the
problems of the future
(c) The sound arguments about the uncertainty in one's imagination about
humanity's plans for its future
(d) Inclination of the author to promote optimistic solutions for the various problems
humanity will face eventually

Q107) What does the author mean by, "Historical time flows for everyone"?
(a) The facts of changes and events in time are relevant to everyone and no one can avoid
it.

Page 47 of 53
(b) Time is not relative to the experiences of an individual in the long run.
(c) Time is unaffected by individual pursuits.
(d) Time does not stop for anyone, and that has been proven historically.

Q108) What can be inferred when the author says that the future will come for everyone, but
not everyone will hold an equal place in it?
(a) Not everyone deserves equal access to opportunities in the future.
(b) A multi-polar world would serve as a major cause of trouble for some people in the
future.
(c) The plight of some people would be less fortunate than others in the future but they
can't ignore the future.
(d) The level of financial inequality will rise in the future and it
cannot be contained.

SECTION 5 - QUANTITATIVE TECHNIQUES

Directions (Q109-Q120): Read the information from the passages carefully and Solve
the questions.

Passage 1 (Q109-Q113):

Directions : Study the following pie chart and bar chart carefully and answer the questions
given beside.
A person covers a total distance of 3000 km in 6 days, by bus, by Ola, and by Uber. Study the
pie chart and bar graph to answer these questions.

The percentage of total distance traveled in 6 different days.

Page 48 of 53
The percentage of distance covered by each mode on each day.

Q109) What is the total distance covered by Bus?


a) 814.9 km
b) 812.3 km
c) 813.3 km
d) 821.23 km
e) None of these

Page 49 of 53
Q110) If everywhere Ola maintains an average speed of 40 km per hour then what is the total
time (In hour) in six days spent on Ola? (approximately)
a) 25 hours
b) 26 hours
c) 27 hours
d) 28 hours
e) 28.5 hours

Q111) The distance traveled by Ola is approximately how much percentage of the distance
traveled by Uber? (round off two decimal)
a) 97.21%
b) 102.32%
c) 98.34%
d) 99.91%
e) 106.29%

Q112) Find the respective ratio of the distance traveled by Bus, Ola, and Uber.
a) 2711 : 3614 : 3675
b) 8133 : 10837 : 11020
c) 2713 : 3612 : 3675
d) 2717 : 3614 : 3681
e) None of these

Q113) Suppose, Instead of Bus, the person uses Ola and the speed of Ola is 25% more than
the speed of the bus , then approximately how many hours would the person save? (The
average speed of bus is 30 km per hour)
a) 6.8 hours
b) 7.2 hours
c) 4.6 hours
d) 5.4 hours
e) 4.8 hours

Passage 2 (Q114-Q117):

Study the following graph carefully & answer the questions given below it.

Imports of 3 companies over the years (in Rs. Crores)

Page 50 of 53
Q114) In which of the following years, the imports made by company 'A' was exactly equal
to the average import made by it over the given years?
a) 1995
b) 1996
c) 1997
d) B and C
e) None of these

Q115) What was the percentage increase in imports by company ‘B’ from 1995 to 1999?
a) 40
b) 250
c) 100
d) 200
e) None of these

Q116) Imports by company ‘C’ in 1997 is how many more than the average imports by all
companies in the same year?
a) 25
b) 15
c) 35
d) 60
e) None of these

Page 51 of 53
Q117) In which of the following years were the imports made by company ‘A’ exactly half
of the total imports made by company ‘B’ and ‘C’ together in that year?
a) 1994
b) 1996
c) 1998
d) 1995
e) None of these

Passage 3 (Q118-Q120):

Study the following bar and table chart carefully and answer the questions given beside.

Four companies Asus, Dell, Acer and HP sell Laptops, Desktops and Printers.

The chart given below shows the number of Laptops and Desktops sold by each company.

The table given below shows the printers sold as a percentage of the sum of Desktops,
Laptops and Printers sold by each company.

Company Printers

Dell 15%

Acer 18%

Asus 26%

Page 52 of 53
HP 28%

Q118) The total number of printers sold by Dell is what percent of total number of printers
sold by Acer?
a) 45.55%
b) 53.33%
c) 62.5%
d) 48.75%
e) None of these

Q119) What is the average number of products (Laptops, Desktops & Printers) sold by Acer
and Asus?
a) 785
b) 765
c) 775
d) 780
e) 795

Q120) What is the difference between the total laptops sold by Dell, Acer and Asus and the
total Desktops sold by Acer, Asus and HP?
a) 421
b) 429
c) 389
d) 431
e) None of these

Page 53 of 53

You might also like